Solve Repulsive Central Field Integration Problem for Deflection Angle

  • Thread starter Thread starter Pyroadept
  • Start date Start date
  • Tags Tags
    Integration
Click For Summary
The discussion focuses on solving a mechanics problem related to finding the deflection angle of a particle in a repulsive central field defined by the potential U = α/r². The user has attempted to apply the formula for deflection angle but is struggling with the integration process, specifically in transforming the integral to the required form. They have factored out a term and substituted values but are unsure how to simplify the expression involving (ρ² + α/E). A suggestion is made to use a trigonometric substitution to facilitate the integration. The conversation emphasizes the importance of integration techniques in solving the problem.
Pyroadept
Messages
82
Reaction score
0

Homework Statement


Hi, I know this is a mechanics question, but I don't think the actual problem I have with it involves any mechanics, it's just integration techniques.

Find the deflection angle of a particle moving in the following repulsive central field:

U = α/r², α > 0




Homework Equations



Use the formula \int1/(x√(x² - 1))dx = π/2 (π = pi)
where the integral limits are 1 (lower) and ∞ (upper)


The Attempt at a Solution


Hi everyone, here's what I've done so far:

I use the formula χ = | π - 2ϕ_0 |, where χ is the angle of deflection

and then ϕ_0 = \int (ρ/r²√(1 - ρ²/r² - U(r)/E) dr

where the integral limits are r_min (lower) and ∞ (upper)

I am trying to turn this into the form given in the question to apply the formula.

First I factor out a 1/r from inside the square root and sub in the value for U(r):

ϕ_0 = \int (ρ/r√(r² - (ρ² + α/E)) dr


But this is where I get stuck, as I can't see how to turn the (ρ² + α/E) into a 1. Can anyone please point me in the right direction?

Thanks in advance for any help! :)
 
Physics news on Phys.org
Hi Pyroadept! :smile:

(have an integral: ∫ :wink:)

Substitute r = [√(r² - (ρ² + α/E))]s :wink:

(btw, a simple trig substitution will give you that integral anyway)
 
Question: A clock's minute hand has length 4 and its hour hand has length 3. What is the distance between the tips at the moment when it is increasing most rapidly?(Putnam Exam Question) Answer: Making assumption that both the hands moves at constant angular velocities, the answer is ## \sqrt{7} .## But don't you think this assumption is somewhat doubtful and wrong?

Similar threads

  • · Replies 3 ·
Replies
3
Views
3K
  • · Replies 4 ·
Replies
4
Views
2K
Replies
5
Views
2K
Replies
3
Views
1K
  • · Replies 7 ·
Replies
7
Views
2K
Replies
5
Views
2K
Replies
6
Views
2K
  • · Replies 3 ·
Replies
3
Views
2K
  • · Replies 5 ·
Replies
5
Views
3K
  • · Replies 5 ·
Replies
5
Views
3K